K
Khách

Hãy nhập câu hỏi của bạn vào đây, nếu là tài khoản VIP, bạn sẽ được ưu tiên trả lời.

25 tháng 6 2022

-983

25 tháng 6 2022

3+78+77+90-667-564 =-983

 

6 tháng 6 2019

Lời giải chi tiết:

47 > 45 34 < 50 10 = 18 – 8
81 < 82 78 > 69 44 > 30 + 10
95 > 90 72 < 81 70 = 90 – 20
61 < 63 62 > 26 77 < 90 – 10
18 tháng 5 2019

a) 89 + 10 x 2 = 89 + 20

                   = 109

b) 25 x 2 + 78 = 50 + 78

                   = 128

c) 46 + 77 x 2 = 46 + 14

                   = 60

d) 35 x 2 + 90 = 70 + 90

                   = 160

 

28 tháng 12 2016

đề bài, tui ko còn giữ sách lớp 6 đâu
 

28 tháng 12 2016

cho n là số tự nhiên

a,   (n+ 10) (n+ 15) chia hết cho 2

b,    n (n+ 1) (n+2) chia hết cho 2 và 3

c,     n (n+ 1) (2n+1) chia hết cho 2 và 3

26 tháng 7 2019

34 < 50     47 > 45     55 < 66

78 > 69     81< 82     44 > 33

72 < 81     95 > 90     77 < 99

62 = 62     61 < 63     88 > 22

=>(1/2+1/3+...+1/80)*x>(1+1/79+1+2/78+...+1+78/2+1)

=>\(x\left(\dfrac{1}{2}+\dfrac{1}{3}+...+\dfrac{1}{80}\right)>\dfrac{80}{80}+\dfrac{80}{79}+...+\dfrac{80}{3}+\dfrac{80}{2}\)

=>x>80

24 tháng 6 2018

a/ là 8

b/ là 3615924564

HỌC TỐT NHÉ 

NHỚ CHỌN CHO MÌNH NHÉ

24 tháng 6 2018

Bạn ghi cách làm giúp mình được không

28 tháng 4 2019

Định lí Py-ta-go : Xét tam giác vuông có độ dài các cạnh góc vuông là a;b và cạnh huyền là c thì ta có

      \(a^2+b^2=c^2\)

Và ngược lại , nếu có hệ thức trên thì tam giác đó cũng vuông

Bài kia : 

Ta có tổng quát \(\frac{1}{\left(n+1\right)\sqrt{n}+n\sqrt{n+1}}=\frac{1}{\sqrt{n\left(n+1\right)}\left(\sqrt{n+1}+\sqrt{n}\right)}\)

                                                                              \(=\frac{\sqrt{n+1}-\sqrt{n}}{\sqrt{n\left(n+1\right)}\left(n+1-n\right)}\)

                                                                             \(=\frac{\sqrt{n+1}-\sqrt{n}}{\sqrt{n\left(n+1\right)}}\)

                                                                              \(=\frac{1}{\sqrt{n}}-\frac{1}{\sqrt{n+1}}\)

Áp dụng ta được

\(H=\frac{1}{1}-\frac{1}{\sqrt{2}}+\frac{1}{\sqrt{2}}-\frac{1}{\sqrt{3}}+\frac{1}{\sqrt{3}}-....+\frac{1}{\sqrt{77}}-\frac{1}{\sqrt{78}}\)

      \(=1-\frac{1}{\sqrt{78}}\)

6 tháng 8 2018

-879 + [64 + (-879) + 36]

= -879 + 64 - 879 + 36

= (-879 - 879) + (64 + 36)

= -1768 + 100 = -1668

-564 + [(-724) + 564 + 224

= -564 -724 + 564 + 224

= (-564 + 564) + (-724 + 224)

= 0 - 500 = - 500